Proof using AM-GM inequality











up vote
2
down vote

favorite












The questions has two parts:



Prove



(i) $ xy^{3} leq frac{1}{4}x^{4} + frac{3}{4}y^{4} $



and



(ii) $ xy^{3} + x^{3}y leq x^{4} + y^{4}$.



Now then, I went about putting both sides of $sqrt{xy} leq frac{1}{2}(x+y)$
to the power of 4 and it left me with



$$-x^{3}y leq frac{1}{4}x^{4} + frac{1}{4}y^{4} + xy^{3} + frac{5}{2}x^{2}y^{2}. $$



Curiously squaring and multiplying $sqrt{xy} leq frac{1}{2}(x+y)$ I've tried merging the results with my other inequality a few times to no avail - I just can't seem to get the signs right and nor can I seem to make the coefficients of the $x^{4}$ and $y^{4}$ different, as they are in (i). I feel like there's something I'm missing. Does anyone see a nice way about this problem?










share|cite|improve this question
























  • Surely, if you get the first, you can get the second. However, is your version of AM-GM limited to just $sqrt{xy}leq1/2(x+y)$?
    – Robert Wolfe
    Nov 30 at 16:41










  • Yes, the book only allows us to assume $sqrt{xy} leq frac{1}{2}(x+y)$. The thing is, I can't get the first...
    – Francis
    Nov 30 at 16:43






  • 1




    This could be killed in a single line with Young's Inequality or the full power of the AM-GM inequality. However, since you're limited to just this special case, this will require more care. You'll probably either want to take an algebraic approach and find a polynomial that is definitely positive (like how one gets your inequality from $(x-y)^2geq 0$) or you'll want to take advantage of the fact that your inequality is most helpful when $x$ and $y$ are close together.
    – Robert Wolfe
    Nov 30 at 16:57












  • @RobertWolfe, thank you - but I'm not sure I see how $x$ and $y$ being close together could be helpful to me?
    – Francis
    Nov 30 at 17:01

















up vote
2
down vote

favorite












The questions has two parts:



Prove



(i) $ xy^{3} leq frac{1}{4}x^{4} + frac{3}{4}y^{4} $



and



(ii) $ xy^{3} + x^{3}y leq x^{4} + y^{4}$.



Now then, I went about putting both sides of $sqrt{xy} leq frac{1}{2}(x+y)$
to the power of 4 and it left me with



$$-x^{3}y leq frac{1}{4}x^{4} + frac{1}{4}y^{4} + xy^{3} + frac{5}{2}x^{2}y^{2}. $$



Curiously squaring and multiplying $sqrt{xy} leq frac{1}{2}(x+y)$ I've tried merging the results with my other inequality a few times to no avail - I just can't seem to get the signs right and nor can I seem to make the coefficients of the $x^{4}$ and $y^{4}$ different, as they are in (i). I feel like there's something I'm missing. Does anyone see a nice way about this problem?










share|cite|improve this question
























  • Surely, if you get the first, you can get the second. However, is your version of AM-GM limited to just $sqrt{xy}leq1/2(x+y)$?
    – Robert Wolfe
    Nov 30 at 16:41










  • Yes, the book only allows us to assume $sqrt{xy} leq frac{1}{2}(x+y)$. The thing is, I can't get the first...
    – Francis
    Nov 30 at 16:43






  • 1




    This could be killed in a single line with Young's Inequality or the full power of the AM-GM inequality. However, since you're limited to just this special case, this will require more care. You'll probably either want to take an algebraic approach and find a polynomial that is definitely positive (like how one gets your inequality from $(x-y)^2geq 0$) or you'll want to take advantage of the fact that your inequality is most helpful when $x$ and $y$ are close together.
    – Robert Wolfe
    Nov 30 at 16:57












  • @RobertWolfe, thank you - but I'm not sure I see how $x$ and $y$ being close together could be helpful to me?
    – Francis
    Nov 30 at 17:01















up vote
2
down vote

favorite









up vote
2
down vote

favorite











The questions has two parts:



Prove



(i) $ xy^{3} leq frac{1}{4}x^{4} + frac{3}{4}y^{4} $



and



(ii) $ xy^{3} + x^{3}y leq x^{4} + y^{4}$.



Now then, I went about putting both sides of $sqrt{xy} leq frac{1}{2}(x+y)$
to the power of 4 and it left me with



$$-x^{3}y leq frac{1}{4}x^{4} + frac{1}{4}y^{4} + xy^{3} + frac{5}{2}x^{2}y^{2}. $$



Curiously squaring and multiplying $sqrt{xy} leq frac{1}{2}(x+y)$ I've tried merging the results with my other inequality a few times to no avail - I just can't seem to get the signs right and nor can I seem to make the coefficients of the $x^{4}$ and $y^{4}$ different, as they are in (i). I feel like there's something I'm missing. Does anyone see a nice way about this problem?










share|cite|improve this question















The questions has two parts:



Prove



(i) $ xy^{3} leq frac{1}{4}x^{4} + frac{3}{4}y^{4} $



and



(ii) $ xy^{3} + x^{3}y leq x^{4} + y^{4}$.



Now then, I went about putting both sides of $sqrt{xy} leq frac{1}{2}(x+y)$
to the power of 4 and it left me with



$$-x^{3}y leq frac{1}{4}x^{4} + frac{1}{4}y^{4} + xy^{3} + frac{5}{2}x^{2}y^{2}. $$



Curiously squaring and multiplying $sqrt{xy} leq frac{1}{2}(x+y)$ I've tried merging the results with my other inequality a few times to no avail - I just can't seem to get the signs right and nor can I seem to make the coefficients of the $x^{4}$ and $y^{4}$ different, as they are in (i). I feel like there's something I'm missing. Does anyone see a nice way about this problem?







inequality a.m.-g.m.-inequality geometric-inequalities






share|cite|improve this question















share|cite|improve this question













share|cite|improve this question




share|cite|improve this question








edited Nov 30 at 16:52









Michael Rozenberg

94.5k1588183




94.5k1588183










asked Nov 30 at 16:36









Francis

353




353












  • Surely, if you get the first, you can get the second. However, is your version of AM-GM limited to just $sqrt{xy}leq1/2(x+y)$?
    – Robert Wolfe
    Nov 30 at 16:41










  • Yes, the book only allows us to assume $sqrt{xy} leq frac{1}{2}(x+y)$. The thing is, I can't get the first...
    – Francis
    Nov 30 at 16:43






  • 1




    This could be killed in a single line with Young's Inequality or the full power of the AM-GM inequality. However, since you're limited to just this special case, this will require more care. You'll probably either want to take an algebraic approach and find a polynomial that is definitely positive (like how one gets your inequality from $(x-y)^2geq 0$) or you'll want to take advantage of the fact that your inequality is most helpful when $x$ and $y$ are close together.
    – Robert Wolfe
    Nov 30 at 16:57












  • @RobertWolfe, thank you - but I'm not sure I see how $x$ and $y$ being close together could be helpful to me?
    – Francis
    Nov 30 at 17:01




















  • Surely, if you get the first, you can get the second. However, is your version of AM-GM limited to just $sqrt{xy}leq1/2(x+y)$?
    – Robert Wolfe
    Nov 30 at 16:41










  • Yes, the book only allows us to assume $sqrt{xy} leq frac{1}{2}(x+y)$. The thing is, I can't get the first...
    – Francis
    Nov 30 at 16:43






  • 1




    This could be killed in a single line with Young's Inequality or the full power of the AM-GM inequality. However, since you're limited to just this special case, this will require more care. You'll probably either want to take an algebraic approach and find a polynomial that is definitely positive (like how one gets your inequality from $(x-y)^2geq 0$) or you'll want to take advantage of the fact that your inequality is most helpful when $x$ and $y$ are close together.
    – Robert Wolfe
    Nov 30 at 16:57












  • @RobertWolfe, thank you - but I'm not sure I see how $x$ and $y$ being close together could be helpful to me?
    – Francis
    Nov 30 at 17:01


















Surely, if you get the first, you can get the second. However, is your version of AM-GM limited to just $sqrt{xy}leq1/2(x+y)$?
– Robert Wolfe
Nov 30 at 16:41




Surely, if you get the first, you can get the second. However, is your version of AM-GM limited to just $sqrt{xy}leq1/2(x+y)$?
– Robert Wolfe
Nov 30 at 16:41












Yes, the book only allows us to assume $sqrt{xy} leq frac{1}{2}(x+y)$. The thing is, I can't get the first...
– Francis
Nov 30 at 16:43




Yes, the book only allows us to assume $sqrt{xy} leq frac{1}{2}(x+y)$. The thing is, I can't get the first...
– Francis
Nov 30 at 16:43




1




1




This could be killed in a single line with Young's Inequality or the full power of the AM-GM inequality. However, since you're limited to just this special case, this will require more care. You'll probably either want to take an algebraic approach and find a polynomial that is definitely positive (like how one gets your inequality from $(x-y)^2geq 0$) or you'll want to take advantage of the fact that your inequality is most helpful when $x$ and $y$ are close together.
– Robert Wolfe
Nov 30 at 16:57






This could be killed in a single line with Young's Inequality or the full power of the AM-GM inequality. However, since you're limited to just this special case, this will require more care. You'll probably either want to take an algebraic approach and find a polynomial that is definitely positive (like how one gets your inequality from $(x-y)^2geq 0$) or you'll want to take advantage of the fact that your inequality is most helpful when $x$ and $y$ are close together.
– Robert Wolfe
Nov 30 at 16:57














@RobertWolfe, thank you - but I'm not sure I see how $x$ and $y$ being close together could be helpful to me?
– Francis
Nov 30 at 17:01






@RobertWolfe, thank you - but I'm not sure I see how $x$ and $y$ being close together could be helpful to me?
– Francis
Nov 30 at 17:01












2 Answers
2






active

oldest

votes

















up vote
5
down vote



accepted










Because by AM-GM $$3y^4+x^4geq4sqrt[4]{left(y^4right)^3x^4}=4|xy^3|geq4xy^3$$ and
$$x^4+y^4-x^3y-xy^3=(x-y)(x^3-y^3)=(x-y)^2(x^2+xy+y^2)geq$$
$$geq(x-y)^2left(2sqrt{x^2y^2}+xyright)=(x-y)^2left(2|xy|+xyright)geq0.$$






share|cite|improve this answer






























    up vote
    2
    down vote













    (i) follows from Young's inequality for $p = 4$ and $q = frac43$:



    $$xy^3 le |x||y|^3 le frac{|x|^p}{p} + frac{left(|y|^3right)^q}q = frac{|x|^4}4 + frac{left(|y|^3right)^{4/3}}{4/3} = frac{|x|^4+3|y|^4}4 = frac{x^4+3y^4}4$$



    (ii) follows from (i):



    $$xy^3 + x^3y le frac{x^4+3y^4}4 + frac{3x^4+y^4}4 = x^4+y^4$$






    share|cite|improve this answer























    • @MichaelRozenberg Good catch, thanks. Corrected now.
      – mechanodroid
      Nov 30 at 17:06











    Your Answer





    StackExchange.ifUsing("editor", function () {
    return StackExchange.using("mathjaxEditing", function () {
    StackExchange.MarkdownEditor.creationCallbacks.add(function (editor, postfix) {
    StackExchange.mathjaxEditing.prepareWmdForMathJax(editor, postfix, [["$", "$"], ["\\(","\\)"]]);
    });
    });
    }, "mathjax-editing");

    StackExchange.ready(function() {
    var channelOptions = {
    tags: "".split(" "),
    id: "69"
    };
    initTagRenderer("".split(" "), "".split(" "), channelOptions);

    StackExchange.using("externalEditor", function() {
    // Have to fire editor after snippets, if snippets enabled
    if (StackExchange.settings.snippets.snippetsEnabled) {
    StackExchange.using("snippets", function() {
    createEditor();
    });
    }
    else {
    createEditor();
    }
    });

    function createEditor() {
    StackExchange.prepareEditor({
    heartbeatType: 'answer',
    convertImagesToLinks: true,
    noModals: true,
    showLowRepImageUploadWarning: true,
    reputationToPostImages: 10,
    bindNavPrevention: true,
    postfix: "",
    imageUploader: {
    brandingHtml: "Powered by u003ca class="icon-imgur-white" href="https://imgur.com/"u003eu003c/au003e",
    contentPolicyHtml: "User contributions licensed under u003ca href="https://creativecommons.org/licenses/by-sa/3.0/"u003ecc by-sa 3.0 with attribution requiredu003c/au003e u003ca href="https://stackoverflow.com/legal/content-policy"u003e(content policy)u003c/au003e",
    allowUrls: true
    },
    noCode: true, onDemand: true,
    discardSelector: ".discard-answer"
    ,immediatelyShowMarkdownHelp:true
    });


    }
    });














    draft saved

    draft discarded


















    StackExchange.ready(
    function () {
    StackExchange.openid.initPostLogin('.new-post-login', 'https%3a%2f%2fmath.stackexchange.com%2fquestions%2f3020293%2fproof-using-am-gm-inequality%23new-answer', 'question_page');
    }
    );

    Post as a guest















    Required, but never shown

























    2 Answers
    2






    active

    oldest

    votes








    2 Answers
    2






    active

    oldest

    votes









    active

    oldest

    votes






    active

    oldest

    votes








    up vote
    5
    down vote



    accepted










    Because by AM-GM $$3y^4+x^4geq4sqrt[4]{left(y^4right)^3x^4}=4|xy^3|geq4xy^3$$ and
    $$x^4+y^4-x^3y-xy^3=(x-y)(x^3-y^3)=(x-y)^2(x^2+xy+y^2)geq$$
    $$geq(x-y)^2left(2sqrt{x^2y^2}+xyright)=(x-y)^2left(2|xy|+xyright)geq0.$$






    share|cite|improve this answer



























      up vote
      5
      down vote



      accepted










      Because by AM-GM $$3y^4+x^4geq4sqrt[4]{left(y^4right)^3x^4}=4|xy^3|geq4xy^3$$ and
      $$x^4+y^4-x^3y-xy^3=(x-y)(x^3-y^3)=(x-y)^2(x^2+xy+y^2)geq$$
      $$geq(x-y)^2left(2sqrt{x^2y^2}+xyright)=(x-y)^2left(2|xy|+xyright)geq0.$$






      share|cite|improve this answer

























        up vote
        5
        down vote



        accepted







        up vote
        5
        down vote



        accepted






        Because by AM-GM $$3y^4+x^4geq4sqrt[4]{left(y^4right)^3x^4}=4|xy^3|geq4xy^3$$ and
        $$x^4+y^4-x^3y-xy^3=(x-y)(x^3-y^3)=(x-y)^2(x^2+xy+y^2)geq$$
        $$geq(x-y)^2left(2sqrt{x^2y^2}+xyright)=(x-y)^2left(2|xy|+xyright)geq0.$$






        share|cite|improve this answer














        Because by AM-GM $$3y^4+x^4geq4sqrt[4]{left(y^4right)^3x^4}=4|xy^3|geq4xy^3$$ and
        $$x^4+y^4-x^3y-xy^3=(x-y)(x^3-y^3)=(x-y)^2(x^2+xy+y^2)geq$$
        $$geq(x-y)^2left(2sqrt{x^2y^2}+xyright)=(x-y)^2left(2|xy|+xyright)geq0.$$







        share|cite|improve this answer














        share|cite|improve this answer



        share|cite|improve this answer








        edited Nov 30 at 16:50

























        answered Nov 30 at 16:43









        Michael Rozenberg

        94.5k1588183




        94.5k1588183






















            up vote
            2
            down vote













            (i) follows from Young's inequality for $p = 4$ and $q = frac43$:



            $$xy^3 le |x||y|^3 le frac{|x|^p}{p} + frac{left(|y|^3right)^q}q = frac{|x|^4}4 + frac{left(|y|^3right)^{4/3}}{4/3} = frac{|x|^4+3|y|^4}4 = frac{x^4+3y^4}4$$



            (ii) follows from (i):



            $$xy^3 + x^3y le frac{x^4+3y^4}4 + frac{3x^4+y^4}4 = x^4+y^4$$






            share|cite|improve this answer























            • @MichaelRozenberg Good catch, thanks. Corrected now.
              – mechanodroid
              Nov 30 at 17:06















            up vote
            2
            down vote













            (i) follows from Young's inequality for $p = 4$ and $q = frac43$:



            $$xy^3 le |x||y|^3 le frac{|x|^p}{p} + frac{left(|y|^3right)^q}q = frac{|x|^4}4 + frac{left(|y|^3right)^{4/3}}{4/3} = frac{|x|^4+3|y|^4}4 = frac{x^4+3y^4}4$$



            (ii) follows from (i):



            $$xy^3 + x^3y le frac{x^4+3y^4}4 + frac{3x^4+y^4}4 = x^4+y^4$$






            share|cite|improve this answer























            • @MichaelRozenberg Good catch, thanks. Corrected now.
              – mechanodroid
              Nov 30 at 17:06













            up vote
            2
            down vote










            up vote
            2
            down vote









            (i) follows from Young's inequality for $p = 4$ and $q = frac43$:



            $$xy^3 le |x||y|^3 le frac{|x|^p}{p} + frac{left(|y|^3right)^q}q = frac{|x|^4}4 + frac{left(|y|^3right)^{4/3}}{4/3} = frac{|x|^4+3|y|^4}4 = frac{x^4+3y^4}4$$



            (ii) follows from (i):



            $$xy^3 + x^3y le frac{x^4+3y^4}4 + frac{3x^4+y^4}4 = x^4+y^4$$






            share|cite|improve this answer














            (i) follows from Young's inequality for $p = 4$ and $q = frac43$:



            $$xy^3 le |x||y|^3 le frac{|x|^p}{p} + frac{left(|y|^3right)^q}q = frac{|x|^4}4 + frac{left(|y|^3right)^{4/3}}{4/3} = frac{|x|^4+3|y|^4}4 = frac{x^4+3y^4}4$$



            (ii) follows from (i):



            $$xy^3 + x^3y le frac{x^4+3y^4}4 + frac{3x^4+y^4}4 = x^4+y^4$$







            share|cite|improve this answer














            share|cite|improve this answer



            share|cite|improve this answer








            edited Nov 30 at 17:06

























            answered Nov 30 at 16:44









            mechanodroid

            25k62245




            25k62245












            • @MichaelRozenberg Good catch, thanks. Corrected now.
              – mechanodroid
              Nov 30 at 17:06


















            • @MichaelRozenberg Good catch, thanks. Corrected now.
              – mechanodroid
              Nov 30 at 17:06
















            @MichaelRozenberg Good catch, thanks. Corrected now.
            – mechanodroid
            Nov 30 at 17:06




            @MichaelRozenberg Good catch, thanks. Corrected now.
            – mechanodroid
            Nov 30 at 17:06


















            draft saved

            draft discarded




















































            Thanks for contributing an answer to Mathematics Stack Exchange!


            • Please be sure to answer the question. Provide details and share your research!

            But avoid



            • Asking for help, clarification, or responding to other answers.

            • Making statements based on opinion; back them up with references or personal experience.


            Use MathJax to format equations. MathJax reference.


            To learn more, see our tips on writing great answers.





            Some of your past answers have not been well-received, and you're in danger of being blocked from answering.


            Please pay close attention to the following guidance:


            • Please be sure to answer the question. Provide details and share your research!

            But avoid



            • Asking for help, clarification, or responding to other answers.

            • Making statements based on opinion; back them up with references or personal experience.


            To learn more, see our tips on writing great answers.




            draft saved


            draft discarded














            StackExchange.ready(
            function () {
            StackExchange.openid.initPostLogin('.new-post-login', 'https%3a%2f%2fmath.stackexchange.com%2fquestions%2f3020293%2fproof-using-am-gm-inequality%23new-answer', 'question_page');
            }
            );

            Post as a guest















            Required, but never shown





















































            Required, but never shown














            Required, but never shown












            Required, but never shown







            Required, but never shown

































            Required, but never shown














            Required, but never shown












            Required, but never shown







            Required, but never shown







            Popular posts from this blog

            QoS: MAC-Priority for clients behind a repeater

            Ивакино (Тотемский район)

            Can't locate Autom4te/ChannelDefs.pm in @INC (when it definitely is there)